Domain Of K In $f(x)=x^3+kx^2+5x+4\sin^2x - Math Stack Exchange
Có thể bạn quan tâm
-
- Home
- Questions
- Tags
- Users
- Unanswered
- Teams
Ask questions, find answers and collaborate at work with Stack Overflow for Teams.
Try Teams for free Explore Teams - Teams
-
Ask questions, find answers and collaborate at work with Stack Overflow for Teams. Explore Teams
Teams
Q&A for work
Connect and share knowledge within a single location that is structured and easy to search.
Learn more about Teams Domain of k in $f(x)=x^3+kx^2+5x+4\sin^2x$ Ask Question Asked 6 years, 10 months ago Modified 6 years, 10 months ago Viewed 301 times 4 $\begingroup$The question says:
Let $f(x)=x^3+kx^2+5x+4\sin^2x$ be an increasing function on $x \in R$. Then domain of k is?
This is What I tried:
I tried differentiating the expression but $f'(x)$ didn't turn out to be a quadratic(as it has that $\sin2x$ term) $$f'(x)=3x^2+2kx+5+4\sin2x>0$$
So how else am I supposed to solve this question?
Share Cite Follow edited Feb 26, 2018 at 1:19 user99914 asked Feb 24, 2018 at 12:25 CarrickCarrick 1979 bronze badges $\endgroup$ 5- $\begingroup$ @Netchaiev wolframalpha.com/input/?i=derivative+sin%5Bx%5D%5E2 $\endgroup$ – GNUSupporter 8964民主女神 地下教會 Commented Feb 24, 2018 at 12:35
- $\begingroup$ @Netchaiev I know its $2\sin x \cos x$ but aren't they both the same?? $\endgroup$ – Carrick Commented Feb 24, 2018 at 12:35
- $\begingroup$ yes you are both right, my bad :) (deleting the first comment) $\endgroup$ – Netchaiev Commented Feb 24, 2018 at 12:37
- $\begingroup$ @Netchaiev not a big deal..its ok $\endgroup$ – Carrick Commented Feb 24, 2018 at 12:38
- $\begingroup$ @Netchaiev It's alright. $\endgroup$ – GNUSupporter 8964民主女神 地下教會 Commented Feb 24, 2018 at 12:39
2 Answers
Sorted by: Reset to default Highest score (default) Date modified (newest first) Date created (oldest first) 2 $\begingroup$The set up of the problem is completely right with only one remark: you can set $$f'(x)\ge0$$ to have $f(x)$ increasing and maybe also strictly increasing since the the set on which $f'(x)=0$ for "regular functions" like the given doesn't contain any intervals.
You can refer also to this other answer for a detailed comment on this latter fact.
For a first estimation of the range note that for
$$3x^2+2kx+5\ge4$$
the condition is satisfy, thus from here we can find a first range for $k\in[-\sqrt 3,\sqrt 3]$, indeed
$$3x^2+2kx+1\ge0 \iff \Delta=4k^2-12\le 0$$
To improve the result we could try to find the minimum of $f'(x)$ and set it to $0$, that is
$$f''(x)=6x+2k+8\cos2x=0 \implies x=x_k$$
and
$$f'(x_k)=0$$
Share Cite Follow edited Feb 24, 2018 at 13:24 answered Feb 24, 2018 at 12:39 useruser 160k13 gold badges84 silver badges154 bronze badges $\endgroup$ 8- 1 $\begingroup$ I thought about that but won't it be wrong because we are fixing the value of x so that sin2x attains maxima or minima?? $\endgroup$ – Carrick Commented Feb 24, 2018 at 12:43
- $\begingroup$ It is a first good estimation since $4\sin2x\ge -4$ if $3x^2+2kx+5\ge4$ we are sure that $f'(x)\ge 0$. . $\endgroup$ – user Commented Feb 24, 2018 at 12:45
- $\begingroup$ sorry...didn't really get it...how does that answer my question?? $\endgroup$ – Carrick Commented Feb 24, 2018 at 12:51
- $\begingroup$ @Carrick No it doesn't answer it is only an hint, I'm thinking about it to improve the estimation and some one else will give other answers, wait to accept as solved, but you can upvote if you think it was useful to you. $\endgroup$ – user Commented Feb 24, 2018 at 12:53
- $\begingroup$ by question I didn't mean the actual queston..I was talking about what I just asked in the comments(fixing the value of x thing) and your hint is correct..it matches with the actual answer $\endgroup$ – Carrick Commented Feb 24, 2018 at 12:59
From $f '(x)=3x^2+2kx+5+4\sin2x$, try to solve $f'(x)=0$, so $$k=\frac{-3x^2-5-4\sin2x}{2x}$$ If you can't solve $f'(x)=0$, then $f(x)$ is increasing, so the domain of $k$ is everything outside the range of that function.
Share Cite Follow answered Feb 26, 2018 at 2:37 Empy2Empy2 51.9k1 gold badge45 silver badges95 bronze badges $\endgroup$ Add a comment |You must log in to answer this question.
Not the answer you're looking for? Browse other questions tagged
.- Featured on Meta
- The December 2024 Community Asks Sprint has been moved to March 2025 (and...
- Stack Overflow Jobs is expanding to more countries
Linked
0 g' strictly increasing different proofRelated
1 Solving an integral using Leibnitz rule 0 derivative of $x\cdot|\sin x|$ 3 Solution for the integral $∫ \sin(4x)(1+\cos(4x))\mathrm dx$ 2 Monotonicity of $f(x) =\sin(\ln(x))-\cos(\ln(x))$ 0 Multiplicity of root for $x^2 \sin(x^2) $ 0 Nth derivative of the function y=$e^{ax}\sin{bx}$ 1 let $F(X)=\int_0^x \sin\frac{1}{1+t^2}\,dt$, then which of these is correct? 1 What is the maximum and minimum value of the following function?Hot Network Questions
- How was Lemech allowed to hunt?
- Time served and logical propositions
- Auto-configuring Global Unicast address with prefixed other than 64-bits len
- Help in identifying this dot-sized insect crawling on my bed
- How do I find the right size cap to close a water valve?
- Study the convergence of improper integral from 0 to infinity
- Would it be possible to use a Cygnus resupply spacecraft as a temporary space station?
- Ways to keep files in /tmp?
- How to merge at intersection line point?
- How to account for disproportionate group sizes?
- I probably disallowed using the camera at some time in the past and now can't find a way to allow it again. How can I reenable it?
- Model structure on commutative monoids
- C++20 Robust File Interface
- Misunderstanding a code
- How to repair a broken ground wire?
- Bolt of rear derailleur rounded out and broke off - repair wire thread
- Best Practices for Managing Open-Source Vulnerabilities in Enterprise Deployments
- Can a 4-d creature twist your right hand into a left hand without breaking it?
- Physically/fundamentally/literally, why does a significant voltage not develop across a diode in forward bias?
- Reference request for an Einstein quote
- PSE Advent Calendar 2024 (Day 21): Wrap-Up
- What does "No Reachable Authority at delegation" mean? (aka NS record not returned by the DNS)
- Does "To the Moon" generate interest while using the Green Deck?
- A mistake in revised version
To subscribe to this RSS feed, copy and paste this URL into your RSS reader.
Từ khóa » F(x)=x^3+kx^2-4x-12
-
F(x)=x^3+kx^2-4x-12 And G(a)=a+a^3+ A^6÷a^3 Hollya A){x€N:x^2>
-
If X+2 Is A Factor Of Fx=x3+kx2-4x+12 Then Find Th - Gauthmath
-
If X-2 Is A Factor Of X^3-kx^2+kx+2, Where K Is A Constant, What Is The ...
-
If F(x)=x^(3)-kx^(2)+2x And F(-x)= -f(x), The Value Of K Is - Doubtnut
-
Find The Value Of K , If X - 2 Is A Factor Of Polynomial P(x) = X^3 + Kx^2 ...
-
Remainder Theorem: Find K If F(x)=x^2+kx-20 - YouTube
-
[PDF] 3.2 The Factor Theorem And The Remainder Theorem
-
[PDF] Remainder & Factor Theorems
-
When The Function F(x) =x^3+kx^2-7x+3 Is Divided By (x+1 ... - Quora
-
SOLUTION: Find K Such That F(x)=x^3-kx^2+kx+2 Has The Factor X-2. I ...